If you find any mistakes, please make a comment! Thank you.

Solutions to Real Analysis: A Long-Form Mathematics Textbook Chapter 4


Exercise 4.1

Solution:

(1) Converges conditionally. It is convergent by Alternating Series Test. But $\sum_{k=1}^\infty\dfrac{1}{\sqrt{k}}$ is divergent by Proposition 4.16.

(2) Converges absolutely. By Geometric Series Test and the fact that $\ln 4 > 1$.

(3) Diverges. Note that \[\sqrt{n+1}-\sqrt n=\frac{(n+1)-n}{\sqrt{n+1}+\sqrt{n}} > \frac{1}{2\sqrt{n+1}}.\]Then use Proposition 4.16 and Comparison Test.

(4) Diverges. By $k^{\rm th}$-term test and the fact that $\lim \dfrac{k}{k+7}=1 \ne 0$.

(5) Converges absolutely. Use the Ratio test in Exercise 4.9.

(6) Diverges. By $k^{\rm th}$-term test.


Exercise 4.2

Solution: Let $s_n$ be the partial sum of the series $\sum_{k=1}^\infty a_k$. Let $S_n$ be the partial sum of the series $\sum_{k=1}^\infty |a_k|$.

Since the series $\sum_{k=1}^\infty |a_k|$ converges, the sequence $(S_n)$ converges and hence is Cauchy. For any $\e >0$, there exists $N$ such that $$|S_{m}-S_n|=\sum_{k=n+1}^m|a_k|$$for all $m>n > N$.By the triangle inequality, we have\[\left|\sum_{k=n+1}^ma_k\right|\leqslant \sum_{k=n+1}^m|a_k|.\]Therefore for all $m > n > N$, we have\begin{align*}|s_m-s_n|&=\left|\sum_{k=n+1}^ma_k\right|\\ &\leqslant \sum_{k=n+1}^m|a_k| < \e.\end{align*}Hence the sequence $(s_n)$ is also Cauchy and thus converges by Theorem 3.42. This shows that the series $\sum_{k=1}^\infty a_k$ converges.


Exercise 4.3

Solution: If $|r| > 1$, then let $|r|=1+a$ where $a >0$. We have\[|r|^n=(1+a)^n=1+ n a+\frac{n(n-1)}{2}a+\cdots > 1+na.\]Hence the sequence $(|r|^n)$ is not bounded, so is $(r^n)$. Hence $(r^n)$ diverges.

If $r=1$, it is clear that $(r^n)$ converges to one. If $r=-1$, it is clear that $(r^n)$ diverges.

If $|r| < 1$, we show that $(r^n)$ converges to $0$. Since $\dfrac{1}{|r|} > 1$, we set $\dfrac{1}{|r|}=1+a$, where $a > 0$.

For any $\e >0$, let $N=\dfrac{1}{a\e}$, then $$\frac{1}{na} < \e$$for all $n> N$. Recall from above that $(1+a)^ n > 1+na$, we have\[|r|^n=\frac{1}{(1+a)^n} < \frac{1}{1+na} < \frac{1}{na} < \e.\]Therefore $(r^n)$ converges to zero.


Exercise 4.4

Solution: Consider the harmonic series given by $a_n=\dfrac{1}{n}$. It is known from Proposition 4.15 that $\sum_{k=1}^\infty a_k$ diverges. But the series $$\sum_{k=1}^\infty a_k^2=\sum_{k=1}^\infty \frac{1}{k^2}$$ is convergent by the Series $p$-Test.


Exercise 4.5

Solution:

(a) Since $\sum_{k=1}^\infty a_k$ converges, the sequence $(a_n)$ converges to zero by $k^{\rm th}$-term Test. Hence $(a_n)$ is bounded by Proposition 3.20. Hence there exists $\alpha>0$ such that $a_k < \alpha$ for all $k$. In particular, we have $$0 < a_k^2 < \alpha a_k$$ for all $k$.

Since $\sum_{k=1}^\infty a_k$ converges, so does $\sum_{k=1}^\infty \alpha a_k$ converges. Since $a_k^2 \leqslant \alpha a_k$, by Proposition 4.12 (Comparison test), $\sum_{k=1}^\infty  a_k^2$ converges.

(b) Consider the series $\sum_{k=1}^\infty (-1)^k\frac{1}{\sqrt k}$. This series is convergent by Exercise 1 (1), but $\sum_{k=1}^\infty a_k^2$ is the harmonic series which is divergent.


Exercise 4.6

Solution: Taking the series $\sum_{k=1}^\infty (-1)^k\frac{1}{\sqrt k}$ from Exercise 4.5 (b). Then $\sum_{k=1}^\infty a_k$ converges, but $\sum_{k=1}^\infty a_k^2$ diverges. It is easy to see using Alternating series Test that $\sum_{k=1}^\infty a_k^3$ converges.


Exercise 4.11

Solution: Let $\sum_{k=1}^\infty a_k$ be the harmonic series, then it is divergent. Moreover,\[\lim_{k\to \infty}(a_{k+1}-a_k)=\lim_{k\to \infty}\left(\frac{1}{k+1}-\frac{1}{k}\right)=0.\]


Exercise 4.12

Solution:

(1) Let $a_k=\dfrac{1}{k^2}$ and $b_k=\dfrac{1}{k}$. Then $0\leqslant a_k \leqslant b_k$, $\sum_{k=1}^\infty a_k$ converges, but $\sum_{k=1}^\infty b_k$ diverges.

(2) Let $a_k=\dfrac{1}{k^2}$ and $b_k=\dfrac{1}{k}$. Then $0\leqslant a_k \leqslant b_k$, $\sum_{k=1}^\infty b_k$ diverges, but $\sum_{k=1}^\infty a_k$ converges.


Exercise 4.13

Solution: Let $a_k=-\dfrac{1}{k}$. Since the harmonic series $\sum_{k=1}^\infty \dfrac{1}{k}$ diverges, the series $\sum_{k=1}^\infty a_k$ diverges.

Let $b_k=\dfrac{1}{k^2}$, by the $p$-Series Test, $\sum_{k=1}^\infty b_k$ converges. It is clear that $a_k \leqslant b_k$ for all $k$.


Exercise 4.14

Solution:

(a) This is exactly Exercise 3.19.

(b) Let $a_n=(-1)^{n-1}$. Then $\sum_{k=1}^n a_n$ diverges by $k^{\rm th}$-term Test. We have\[s_{2n-1}=1,\quad s_{2n}=0.\]Hence\[\frac{s_1+\cdots+s_n}{n}=\frac{[n/2]}{n}\]\[\Longrightarrow \frac{n/2-1}{n}\leqslant \frac{s_1+\cdots+s_n}{n} \leqslant \frac{1}{2}.\]Since $\displaystyle\lim_{k\to \infty}\dfrac{k/2-1}{k}=\dfrac{1}{2}$, using the Sequence Squeeze Theorem (Theorem 3.23), we conclude that $$\lim_{n\to \infty}\frac{s_1+\cdots+s_n}{n}=\frac{1}{2}.$$Hence $(a_n)$ is Cesaro summable but divergent.

Linearity

This website is supposed to help you study Linear Algebras. Please only read these solutions after thinking about the problems carefully. Do not just copy these solutions.
Close Menu